Aproximación de funciones por polinomios

Proporcionamos ejercicios de aproximación de funciones por polinomios.

RESUMEN TEÓRICO
  • Como los polinomios son funciones particularmente sencillas, interesará para una función dada $f(x),$ aproximarla por un polinomio $p(x).$ Si $f(x)$ es de clase $n+1$ en un intervalo $[a,b],$ la fórmula de Taylor de orden $n$ en $x_0\in [a,b]$ es: $$f(x)=p(x)+R_n(x)$$ siendo $p(x)$ un polinomio.
  • Al aproximar $f(x)$ mediante el polinomio $p(x),$ el error cometido en valor absoluto es:
    $$\left|R_n(x)\right| = \left|\frac{f^{(n+1)}(\xi)}{(n+1)!} (x-x_0)^{n+1}\right|=\dfrac{\left|f^{(n+1)}(\xi)\right|\;\left| x-x_0\right|^{n+1}}{(n+1)!}$$ en donde $\xi$ está comprendido entre $x_0$ y $x.$ Por hipótesis, $f^{(n+1)}$ es continua en $[a,b]$ y por tanto, también lo es $\left| f^{(n+1)}\right|.$ Por el teorema de Weierstrass, existe máximo absoluto $M$ para ella, es decir se verifica $\left|f^{(n+1)}(\xi)\right|\leq M$ para cualquier $\xi.$ Por tanto, $$\boxed{\;\left|R_n(x)\right| \leq \dfrac{M\left| x-x_0\right|^{n+1}}{(n+1)!}\;}$$
  • La formula anterior nos indica que cuando $x\to x_0$ entonces, $\left|R_n(x)\right|\to 0,$ lo cual nos va a permitir aproximar razonablemente $f(x)$ mediante $p(x)$ en sentido local, es decir en las proximidades de $x_0.$
  • Ejemplo.  Aproximar la función $f(x)=\log (1+x)$ por un polinomio de grado $9$ en el intervalo $[0,1].$ Estimar el error debido a la supresión del resto.
    Vimos en un ejercicio anterior que $$\begin{aligned}\log (1+x)
    =x-\dfrac{x^2}{2}+\dfrac{x^3}{3}-\cdots +\dfrac{(-1)^{n+1}x^n}{n}+\dfrac{(-1)^nx^{n+1}}{(n+1)(\xi +1)^{n+1}},\end{aligned}$$ estando $\xi$ comprendido entre $0$ y $x.$ Para $n=9$ queda: $$\begin{aligned}\log (1+x)
    =x-\dfrac{x^2}{2}+\dfrac{x^3}{3}-\cdots +\dfrac{x^9}{9}-\dfrac{x^{10}}{10(\xi +1)^{10}}.\end{aligned}$$ Acotemos el resto. Teniendo en cuenta que ahora $0<\xi <1:$
    $$\left|-\dfrac{x^{10}}{10(\xi +1)^{10}}\right|=\dfrac{x^{10}}{10(\xi +1)^{10}}< \dfrac{1^{10}}{10(0 +1)^{10}}=\dfrac{1}{10}=0.1.$$
    Enunciado
  1. Acotar el error de la fórmula aproximada: $e\approx 2+\dfrac{1}{2!}+\dfrac{1}{3!}+\dfrac{1}{4!}.$
  2. Calcular aproximadamente $\log 2,$ usando el polinomio de Maclaurin de grado $5$ de la función $f(x)=\log (x+1).$ Dar una cota del error cometido.
  3. Expresar el polinomio $f(x)=x^3-2x^2+6x-7$ en potencias enteras y positivas de $x+1.$
  4. Averiguar cuantos términos hay que tomar en la fórmula de Maclaurin aplicada a la función $f(x)=e^x,$ para obtener un polinomio que la represente en el intervalo $[-1,1],$ con tres cifras decimales exactas.
  5. Determinar un intervalo verificando que la fórmula aproximada $\cos x\approx 1-\dfrac{x^2}{2!}+\dfrac{x^4}{4!}$ tiene un error menor que $0.00005.$
  6. Demostrar que la diferencia entre $\operatorname{sen}(a+h)$ y $\operatorname{sen}a+h\cos a$ no es mayor que $h^2/2.$
  7. Expresar el polinomio $f(x)=x^3-2x^2+3x+5$ en potencias enteras y positivas de $x-2.$
  8. Un hilo pesado, bajo la acción de la gravedad se comba formando la catenaria $y=a\operatorname{ch}\dfrac{x}{a}.$ Demostrar que para valores pequeños de $\left|x\right|,$ la forma que toma el hilo se puede representar aproximadamente por la parábola $y=a+\dfrac{\;x^2}{2a}.$
  9. Calcular $\sqrt{e}$ con un error menor que $10^{-3}.$
  10. $(a)$ Escribir la fórmula de Taylor de orden $3$ en $x_0=1$ para la función $f(x)=\sqrt[3]{x}$
    $(b)$ Siendo $\left|x-1\right|<0.01,$ acotar el error de la fórmula: $$\sqrt[3]{x}\approx \dfrac{5x^3-24x^2+60x+40}{81}.$$
  11. Hallar el polinomio de Maclaurin de grado $4$ de la función $f(x)=\cos x.$ Usando dicho polinomio, hallar un valor aproximado de $\cos 0.1$, dando una cota del error cometido.
    Solución
  1. Aplicando la fórmula de Maclaurin de orden $4$ a la función $f(x)=e^x$ obtenemos: $$e^x=1+\dfrac{x}{1!}+\dfrac{x^2}{2!}+\dfrac{x^3}{3!}+\dfrac{x^4}{4!}+\dfrac{e^{\xi}x^5}{5!}.$$ Para $x=1$ queda: $$e=2+\dfrac{1}{2!}+\dfrac{1}{3!}+\dfrac{1}{4!}+\dfrac{e^{\xi}}{5!},$$ estando $\xi$ entre $0$ y $1,$ es decir $0<\xi<1.$ Acotemos el resto o error: $$\dfrac{e^{\xi}}{5!}<\dfrac{e^{1}}{5!}\leq \dfrac{3}{5!}=\dfrac{1}{40} .$$
  2. Vimos en un ejercicio anterior que $$\begin{aligned}\log (1+x)
    =x-\dfrac{x^2}{2}+\dfrac{x^3}{3}-\cdots +\dfrac{(-1)^{n+1}x^n}{n}+\dfrac{(-1)^nx^{n+1}}{(n+1)(\xi +1)^{n+1}}\end{aligned},$$ estando $\xi$ comprendido entre $0$ y $x.$ Para $n=5$ queda: $$\begin{aligned}\log (1+x)
    =x-\dfrac{x^2}{2}+\dfrac{x^3}{3}-\dfrac{x^4}{4}+\dfrac{x^5}{5}-\dfrac{x^{6}}{6(\xi +1)^{6}}\end{aligned}.$$ Para $x=1:$ $$\begin{aligned}\log 2
    =1-\dfrac{1}{2}+\dfrac{1}{3}-\dfrac{1}{4}+\dfrac{1}{5}-\dfrac{1}{6(\xi +1)^{6}}=\dfrac{47}{60}-\dfrac{1}{6(\xi +1)^{6}}\end{aligned}.$$ Acotemos el resto:
    $$\left|-\dfrac{1}{6(\xi +1)^{6}}\right|=\dfrac{1}{6(\xi +1)^{6}}< \dfrac{1}{6(0 +1)^{6}}=\dfrac{1}{6}.$$ Entonces, $\log 2\approx \dfrac{47}{60}=0.783333\ldots$ con error menor que $\dfrac{1}{6}.$
  3. Apliquemos la fórmula de Taylor a la función $f$ en $x_0=-1:$ $$\begin{aligned}&f(x)=x^3-2x^2+6x-7\Rightarrow f(-1)=-16,\\
    &f'(x)=3x^2-4x+6\Rightarrow f'(-1)=13,\\
    &f^{\prime\prime}(x)=6x-4\Rightarrow f^{\prime\prime}(-1)=-10,\\
    &f^{\prime\prime\prime}(x)=6\Rightarrow f^{\prime\prime\prime}(-1)=6,\\
    &f^{(4)}(x)=0\Rightarrow f^{(4)}(\xi)=0.\end{aligned}$$ Entonces, $$\begin{aligned}f(x) &= f(-1)+ \dfrac{f'(-1)}{1!}(x+1) + \dfrac{f^{(2)}(-1)}{2!}(x+1)^2\\&+\dfrac{f^{(3)}(-1)}{3!}(x+1)^3 + \dfrac{f^{(4)}(\xi)}{4!}(x+1)^4\\
    &=-16+13(x+1)-5(x+1)^2+(x+1)^3.
    \end{aligned}$$
  4. De manera inmediata, deducimos la fórmula de Maclaurin de orden $n$ para $f(x)=e^x:$ $$\begin{aligned}e^x =1+\dfrac{x}{1!}+\dfrac{x^2}{2!}+\dfrac{x^3}{3!}+\cdots+\dfrac{x^n}{n!}+\dfrac{e^{\xi}x^{n+1}}{(n+1)!},\end{aligned}$$ estando $\xi$ comprendido entre $0$ y $x.$ Acotemos el resto en $[-1,1]:$ $$\left|\dfrac{e^{\xi}x^{n+1}}{(n+1)!}\right|=\dfrac{e^{\xi}\left |x\right|^{n+1}}{(n+1)!}\leq \dfrac{e^{1}\cdot 1^{n+1}}{(n+1)!}<\dfrac{3}{(n+1)!}.$$ Para obtener un polinomio que represente a la función en el intervalo $[-1,1]$ con tres cifras decimales exactas, basta que ocurra: $$\dfrac{3}{(n+1)!}<0.001.$$ Ahora bien, $$\dfrac{3}{(n+1)!}<0.001\Leftrightarrow \dfrac{3}{0.001}<(n+1)!\Leftrightarrow30000<(n+1)!.$$ Dando valores a $n=1,2,\ldots,$ verificamos que el primer $n$ que satisface la última desigualdad es $n=6.$
  5. Tenemos $$\begin{aligned}
    &f(x)=\cos x\Rightarrow f(0)=1,\\
    &f'(x)=-\operatorname{sen}x\Rightarrow f'(0)=0,\\
    &f^{\prime\prime}(x)=-\cos x\Rightarrow f»(0)=-1,\\
    &f^{(3)}(x)=\operatorname{sen}x\Rightarrow f^{(3)}(0)=0,\\
    &f^{(4)}(x)=\cos x\Rightarrow f^{(4)}(0)=1,\\
    &f^{(5)}(x)=-\operatorname{sen}x\Rightarrow f^{(5)}(\xi)=-\operatorname{sen}\xi .
    \end{aligned}$$ Por tanto, $$\begin{aligned}\cos x &= f(0)+ \dfrac{f'(0)}{1!}x + \dfrac{f^{(2)}(0)}{2!}x^2+\dfrac{f^{(3)}(0)}{3!}x^3 + \dfrac{f^{(4)}(0)}{4!}x^4+\dfrac{f^{(5)}(\xi)}{5!} x^{5}\\
    &=1-\dfrac{x^2}{2!}+\dfrac{x^4}{4!}-\dfrac{\operatorname{sen}\xi}{5!}x^5,\end{aligned}$$
    estando $\xi$ comprendido entre $0$ y $x.$ Acotando el resto: $$\left|-\dfrac{\operatorname{sen}\xi}{5!}x^5\right|=\dfrac{\left |\operatorname{sen}\xi\right |\left|x\right|^5}{5!}\leq\dfrac{\left|x\right|^5}{5!} .$$ En consecuencia, $$\dfrac{\left|x\right|^5}{5!}<0.00005 \Leftrightarrow \left|x\right|^5<5!\cdot 5\cdot 10^{-5}=6\cdot 10^{-3}\Leftrightarrow \left|x\right|<\sqrt[5]{6\cdot 10^{-3}} ,$$
    y un intervalo que cumple la condición dada es $\left(-\sqrt[5]{6\cdot 10^{-3}},\sqrt[5]{6\cdot 10^{-3}}\right).$
  6. Apliquemos la fórmula de Taylor de orden $2$ en $x_0=a$ a la función $f(x)=\operatorname{sen}x.$ $$\begin{aligned}&f(x)=\operatorname{sen}x\Rightarrow f(a)=\operatorname{sen}a,\\
    & f'(x)=\cos x\Rightarrow f'(a)=\cos a,\\
    &f^{\prime\prime}(x)=-\operatorname{sen}x\Rightarrow f»(\xi)=-\operatorname{sen}\xi.\end{aligned}$$ Entonces, $$\operatorname{sen}x=\operatorname{sen}a+\dfrac{\cos a}{1!}(x-a)-\dfrac{\operatorname{sen}\xi}{2!}(x-a)^2,$$ estando $\xi$ comprendido entre $a$ y $x.$ Llamando $h=x-a$ queda: $$\operatorname{sen}(a+h)-(\operatorname{sen}a+h\cos a)=-\dfrac{h^2\operatorname{sen}\xi}{2},$$ estando $\xi$ comprendido entre $a$ y $a+h.$ Acotando el resto: $$\left|-\dfrac{h^2\operatorname{sen}\xi}{2}\right|=\dfrac{h^2\left|\operatorname{sen}\xi\right|}{2}\leq \dfrac{h^2}{2}.$$
  7. Apliquemos la fórmula de Taylor a la función $f$ en $x_0=2:$ $$\begin{aligned}&f(x)=x^3-2x^2+3x+5\Rightarrow f(2)=11,\\
    &f'(x)=3x^2-4x+3\Rightarrow f'(2)=7,\\
    &f^{\prime\prime}(x)=6x-4\Rightarrow f^{\prime\prime}(2)=8,\\
    &f^{\prime\prime\prime}(x)=6\Rightarrow f^{\prime\prime\prime}(2)=6,\\
    &f^{(4)}(x)=0\Rightarrow f^{(4)}(\xi)=0.\end{aligned}$$ Entonces, $$\begin{aligned}f(x) &= f(2)+ \dfrac{f'(2)}{1!}(x-2) + \dfrac{f^{(2)}(2)}{2!}(x-2)^2\\&+\dfrac{f^{(3)}(2)}{3!}(x-2)^3 + \dfrac{f^{(4)}(\xi)}{4!}(x-2)^4\\
    &=11+7(x-2)+4(x-2)^2+(x-2)^3.
    \end{aligned}$$
  8. Hallemos las derivadas hasta orden $2$ de $y=a\operatorname{ch}\dfrac{x}{a}:$ $$\begin{aligned}
    &y=a\operatorname{ch}\dfrac{x}{a}\Rightarrow y(0)=a,\\
    &y’=\operatorname{sh}\dfrac{x}{a}\Rightarrow y'(0)=0,\\
    &y^{\prime\prime}=\dfrac{1}{a}\operatorname{ch}\dfrac{x}{a}\Rightarrow y^{\prime\prime}(0)=\dfrac{1}{a}.
    \end{aligned}$$ La correspondiente fórmula de Maclaurin para la catenaria es:
    $$y=y(0)+\dfrac{y'(0)}{1!}x+\dfrac{y^{\prime\prime}(0)}{2!}x^2+R_2(y)=a+\dfrac{x^2}{2a}+R_2(y).$$ Sabemos que si $\left|x\right|\to 0,$ entonces $R_2(y)\to 0,$ por tanto para valores pequeños de $\left|x\right|,$ la forma que toma el hilo puede representarse aproximadamemte por la parábola $y=a+\dfrac{\;x^2}{2a}.$
  9. La fórmula de Maclaurin de orden $n$ para $f(x)=e^x$ es: $$\begin{aligned}e^x =1+\dfrac{x}{1!}+\dfrac{x^2}{2!}+\dfrac{x^3}{3!}+\cdots+\dfrac{x^n}{n!}+\dfrac{e^{\xi}x^{n+1}}{(n+1)!},\end{aligned}$$ estando $\xi$ comprendido entre $0$ y $x.$ Si $x=1/2$ obtenemos
    $$\begin{aligned}\sqrt{e} =1+\dfrac{1}{1!\cdot2}+\dfrac{1}{2!\cdot2^2}+\dfrac{1}{3!\cdot 2^3}+\cdots+\dfrac{1}{n!\cdot 2^n}+\dfrac{e^{\xi}}{(n+1)!\cdot 2^{n+1}},\end{aligned}$$
    estando $\xi$ comprendido entre $0$ y $1/2.$ Acotemos el resto: $$\left|\dfrac{e^{\xi}}{(n+1)!\cdot 2^{n+1}}\right|<\dfrac{e^{1/2}}{(n+1)!\cdot 2^{n+1}}\leq \dfrac{e}{(n+1)!\cdot 2^{n+1}}<\dfrac{3}{(n+1)!\cdot 2^{n+1}}.$$ Entonces, $$\dfrac{3}{(n+1)!\cdot 2^{n+1}}<10^{-3}\Leftrightarrow 3000<(n+1)!\cdot 2^{n+1}.$$ Dando valores $n=1,2,\ldots $ en la última desigualdad, verificamos que el primer $n$ que la cumple es $n=4.$ Por tanto: $$\sqrt{e}\approx 1+\dfrac{1}{1!\cdot2}+\dfrac{1}{2!\cdot2^2}+\dfrac{1}{3!\cdot 2^3}+\dfrac{1}{4!\cdot 2^4}=\dfrac{211}{128}=1.6484375,$$ con error menor que $10^{-3}.$
  10. $(a)$ Tenemos: $$\begin{aligned}
    &f(x)=x^{1/3}\Rightarrow f(1)=1,\\
    &f'(x)=\frac{1}{3}x^{-2/3}\Rightarrow f'(1)=\frac{1}{3},\\
    &f^{\prime\prime}(x)=-\frac{2}{9}x^{-5/3}\Rightarrow f^{\prime\prime}(1)=-\frac{2}{9},\\
    &f^{(3)}(x)=\frac{10}{27}x^{-8/3}\Rightarrow f^{(3)}(1)=\frac{10}{27},\\
    &f^{(4)}(x)=-\frac{80}{81}x^{-11/3}\Rightarrow f^{(4)}(\xi)=-\frac{80}{81}\xi^{-11/3},
    \end{aligned}$$ estando $\xi$ comprendido entre $1$ y $x.$ La fórmula de Taylor pedida es: $$\begin{aligned}
    \sqrt[3]{x}=&1+ \dfrac{x-1}{3}- \dfrac{(x-1)^2}{9}+ \dfrac{5(x-1)^3}{81}-\frac{10\xi^{-11/3}(x-1)^4}{243}\\
    &=\dfrac{5x^3-24x^2+60x+40}{81}-\frac{10\xi^{-11/3}(x-1)^4}{243}.
    \end{aligned}$$ $(b)$ La condición $\left|x-1\right|<0.01$ equivale a $0.99<x<1.01,$ com lo cual el valor absoluto del resto se puede acotar en la forma: $$\begin{aligned}
    \left|R_n(x)\right|=\dfrac{10\left|x-1\right|^4}{243\xi^3\sqrt[3]{\xi^2}}<\dfrac{10\cdot 0.01^4}{243\cdot 0.99^3\sqrt[3]{0.99^2}}=4.26971\ldots\times 10^{-10}.
    \end{aligned}$$
  11. Tenemos: $$\begin{aligned}
    &f(x)=\cos x\Rightarrow f(0)=1,\\
    &f'(x)=-\operatorname{sen}x\Rightarrow f'(0)=0,\\
    &f^{\prime\prime}(x)=-\cos x\Rightarrow f^{\prime\prime}(0)=-1,\\
    &f^{(3)}(x)=\operatorname{sen}x\Rightarrow f^{(3)}(0)=0,\\
    &f^{(4)}(x)=\cos x\Rightarrow f^{(4)}(0)=1,\\
    &f^{(5)}(\xi)=-\operatorname{sen}x\Rightarrow f^{(5)}(\xi)=-\operatorname{sen}\xi .
    \end{aligned}$$ Usando la fórmula de Maclaurin, y sustituyendo $x$ por $0.1:$ $$\begin{aligned}
    &\cos x=1-\dfrac{x^2}{2!}+\dfrac{x^4}{4!}-\dfrac{\operatorname{sen}\xi}{5!}x^5\\
    & \cos 0.1=1-\dfrac{0.1^2}{2!}+\dfrac{0.1^4}{4!}-\dfrac{\operatorname{sen}\xi}{5!}0.1^5.
    \end{aligned}$$ Entonces, $\cos 0.1\approx 1-\dfrac{0.1^2}{2!}+\dfrac{0.1^4}{4!}=0.9950041\bar{6}.$ Una cota del error es: $$\left|-\dfrac{\operatorname{sen}\xi}{5!}0.1^5\right|\leq \dfrac{0.1^5}{120}=8.\bar{3}\times 10^{-8}.$$
Esta entrada ha sido publicada en Análisis real y complejo y etiquetada como , , . Guarda el enlace permanente.